2014 dxdy logo

Научный форум dxdy

Математика, Физика, Computer Science, Machine Learning, LaTeX, Механика и Техника, Химия,
Биология и Медицина, Экономика и Финансовая Математика, Гуманитарные науки




Начать новую тему Ответить на тему На страницу Пред.  1, 2
 
 
Сообщение14.11.2006, 22:43 
Заслуженный участник


09/02/06
4382
Москва
Но это доказывает неравенство только в малой окрестностности 0.

 Профиль  
                  
 
 
Сообщение15.11.2006, 01:31 
Заслуженный участник
Аватара пользователя


07/03/06
1898
Москва
Если принять, что в окрестности нуля неравенство выполняется, то функция $tg(sin(x))$ имеет на искомом отрезке точку перегиба, т.е. вторая производная равна нулю. Это приводит к необходимости удовлетворения на искомом отрезке уравнения $2tg(sin(x))cos(x)=tg(x)$. Однако реально похоже всегда $2tg(sin(x))cos(x)>tg(x)$ с много большим зазором, чем изначально.
Добавлено
Мда, точка перегиба есть в диапазоне 0.891 – 0.892, т.е., к сожалению, до $arcsin {\pi/4}=0.902$.

 Профиль  
                  
 
 
Сообщение15.11.2006, 16:11 


21/06/06
1721
Вот еще если везде использовать тангенс половины аргумента, то неравенство можно свести к доказательству того, что

Произведение самих тангенсов на их сумму больше разности, но как доказать это неравенство я пока не знаю.

tg(x)*tg(y)*(tg(x)+tg(y))>tg(x)-tg(y)

Где x>y

 Профиль  
                  
 
 
Сообщение15.11.2006, 20:24 


21/06/06
1721
А можно еще и так рассуждать:

1) Левая часть неравенства: tg(sin(x))=sin(sin(x)) / cos(sin(x)). Ясно что это выражение может только уменьшиться если мы в знаменателе заменим cos(sin(x)) на проосто cos(x). (Используем возрастание синуса и убывание косинуса). Оценка снизу: tg(sin(x)) > sin(sin(x)) / cos(x)

2) Вот для целых m легко по индукции показать, что sin(m*y) < m*sin(y) (Ну конечно, когда sin(y) и cos(y) оба больше нуля). Отсюда имеем, положив y=sin(x), а m=1/cos(x): оценка сверху sin(tg(x)) < sin(sin(x)) / cos(x).

3) Все было бы хорошо, если бы величина 1/cos(x) всегда была целой. Наверно тут как то надо воспользоваться непрерывностью, но как. Уважаемые метры, ну помогите, пожалуйста довести до конца эту задачу.

 Профиль  
                  
 
 
Сообщение15.11.2006, 20:59 
Заслуженный участник
Аватара пользователя


17/10/05
3709
:evil:
Sasha2 писал(а):
Оценка снизу: tg(sin(x)) > sin(sin(x)) / cos(x)

Неверно. При $x \to \pi/2$ имеем: $\tg 1 > +\infty$.

 Профиль  
                  
 
 
Сообщение15.11.2006, 21:38 


21/06/06
1721
Ну откровенно говоря, для значений вплоть до пи/2 это неравенство нет смысла доказывать, его достаточно доказать лишь для значений от нуля до arctg(пи/2). Понятно, что функция в левой части продолжает возрастать, а в правой - уже достигла максимума, а именно своего значения равного 1 (sin(tg(arctg(пи/2)))). А для этого диапазона значений (от нуля до arcttg(пи/2)) данная оценка верная (ну, конечно для тех косинусов, обратные значения которых равны целым числам).

 Профиль  
                  
 
 
Сообщение15.11.2006, 22:09 
Заслуженный участник
Аватара пользователя


07/03/06
1898
Москва
Sasha2 писал(а):
1) Левая часть неравенства: tg(sin(x))=sin(sin(x)) / cos(sin(x)). Ясно что это выражение может только уменьшиться если мы в знаменателе заменим cos(sin(x)) на проосто cos(x). (Используем возрастание синуса и убывание косинуса). Оценка снизу: tg(sin(x)) > sin(sin(x)) / cos(x)

Это неверно и внутри указанного Вами промежутка.
Честно говоря, не видно здесь рассуждений без рядов. Отталкиваясь от сказанного ранее, следует лишь показать, что уравнение $tg(sin(x))=sin(tg(x))$ не имеет решения в указанном промежутке. Т.е. решаем уравнение $x=arctg(arcsin(tg(sin(x))))$. Раскладываем правую часть в ряд $arctg(arcsin(tg(sin(x))))=x+\frac{x^7}{30}+O(x^9)$ видим невозможность решения и справедливость искомого неравенства.

 Профиль  
                  
 
 
Сообщение15.11.2006, 22:29 
Заслуженный участник
Аватара пользователя


01/03/06
13626
Москва
Артамонов Ю.Н. писал(а):
...Раскладываем правую часть в ряд $arctg(arcsin(tg(sin(x))))=x+\frac{x^7}{30}+O(x^9)$ видим невозможность решения и справедливость искомого неравенства.

Наверное, стоит еще раз обратить внимание на один из постов Руста в этой теме:
Цитата:
Но это доказывает неравенство только в малой окрестностности 0.

Разложение по формуле Тейлора с ост. членом в форме Пеано принципиально не дает возможности оценить размер окрестности, в которой будет выполняться какое-либо соотношение с самой функцией, если оно выполняется с ее многочленом Тейлора в этой формуле. А это не есть хорошо. Может быть, пора произнести знаменитые слова из одной комедии Л. Гайдая: "Кац всегда предлагал сдаться"?

 Профиль  
                  
 
 
Сообщение15.11.2006, 22:40 
Заслуженный участник
Аватара пользователя


07/03/06
1898
Москва
Зачем сдаваться в выигранной позиции? :lol:

 Профиль  
                  
 
 
Сообщение15.11.2006, 22:50 
Заслуженный участник
Аватара пользователя


01/03/06
13626
Москва
Значит, я что-то упустил в Ваших рассуждениях, пусть Руст нас рассудит, признав за Вами доказанность неравенства, или отвергнув оное. Я написал, что мне в Вашем д-ве не нравится, но утверждать с уверенностью, что в нем есть лакуна, не буду.

 Профиль  
                  
 
 
Сообщение15.11.2006, 23:30 


21/06/06
1721
Артамонов Ю.Н. писал(а):
Это неверно и внутри указанного Вами промежутка.


Да вижу уже опять знаки перепутал. Ну что ж еще попытаюсь помучить это неравенство.

 Профиль  
                  
 
 
Сообщение16.11.2006, 00:33 
Заслуженный участник
Аватара пользователя


17/10/05
3709
:evil:
Я позволю себе очень неформальное рассуждение. Возможно, оно формализуемо, возможно, нет.
Итак, раасмотрим $f(x) = \arcsin \arctg \sin \tg x$. Эта функция ограничена.

$\sin \tg x \leq 1$, $\Rightarrow$ $\tg \sin x > \sin \tg x$ при $x \in [\arcsin \arctg 1, \pi/2]$. На промежутке $[0,\arctg(\pi/2)]$ обе функции монотонно растут. Откуда (полагая $a_0 = \arcsin \arctg 1$, $b_0 = \pi/2$) имеем соблюдение неравенства на $[a_1,b_1]$, $a_1 = f(a_0)$, $b_1 = f(b_1)$. Заметим, что наши интервалы пересекаются, и в дальнейшем будем рассматривать только $[a_{k+1},a_{k}]$. Повторяя этот процесс, мы получаем монтонно убывающую цепочку $a_k$, для которых неравенство верно.

Дальше я сошлюсь на старые обсуждения (1) и (2) (Руста и прочих). Для $f(x)$ мы имеем разложение в ряд $x -\frac{x^7}{30} + {\rm o}(x^8)$, поэтому $a_k \sim (\frac{5}{k})^{1/6}$. То есть, уверенно (хотя и медленно) стремится к нулю.

 Профиль  
                  
 
 
Сообщение16.11.2006, 01:27 
Модератор
Аватара пользователя


11/01/06
5660
Для тех кто сдался: РЕШЕНИЕ (Problem 3)

 Профиль  
                  
 
 
Сообщение16.11.2006, 02:17 


21/06/06
1721
Ну что ж, не совсем все понятно (но понятно хоть где пробел в знаниях), честно говоря я про такие штучки и не знал. Надо значит выучить про выпуклые функции. Большое спасибо, уважаемый Maxal.

 Профиль  
                  
Показать сообщения за:  Поле сортировки  
Начать новую тему Ответить на тему  [ Сообщений: 29 ]  На страницу Пред.  1, 2

Модераторы: Модераторы Математики, Супермодераторы



Кто сейчас на конференции

Сейчас этот форум просматривают: нет зарегистрированных пользователей


Вы не можете начинать темы
Вы не можете отвечать на сообщения
Вы не можете редактировать свои сообщения
Вы не можете удалять свои сообщения
Вы не можете добавлять вложения

Найти:
Powered by phpBB © 2000, 2002, 2005, 2007 phpBB Group